Now that's a polynomial

Algebra Level 5

Let x 1 < x 2 < x 3 x_1<x_2<x_3 be the three real roots of the equation 2014 x 3 4029 x 2 + 2 = 0 \sqrt{2014}x^3-4029x^2+2=0 . Find x 2 ( x 1 + x 3 ) x_2(x_1+x_3) .


The answer is 2.

This section requires Javascript.
You are seeing this because something didn't load right. We suggest you, (a) try refreshing the page, (b) enabling javascript if it is disabled on your browser and, finally, (c) loading the non-javascript version of this page . We're sorry about the hassle.

2 solutions

Department 8
Jan 5, 2016

Let n = 2014 n=2014 , then the given equation can be rewritten as:

n x 3 ( 1 + 2 n ) x 2 + 2 = n x 3 x 2 2 n x 2 + 2 = x 2 ( n x 1 ) 2 ( n x 2 1 ) = 0 \sqrt{n}x^3 - (1+2n)x^2 + 2 \\ = \sqrt{n}x^3 - x^2 - 2nx^2 + 2 \\ = x^2(\sqrt{n}x - 1) - 2(nx^2 - 1) = 0 .

Note that n x 2 1 = ( n x + 1 ) ( n x 1 ) nx^2-1=( \sqrt{n}x+1)( \sqrt{n}x-1 ) . Now we an factor the last expression as ( n x 1 ) ( x 2 2 ( n x + 1 ) ) = 0 (\sqrt{n}x - 1)(x^2 - 2(\sqrt{n}x+1))=0

This means that 1 n \frac{1}{\sqrt{n}} is a root, and the other two roots are the roots of x 2 2 n x 2 x^2 - 2\sqrt{n}x - 2 . Note that the constant term of the quadratic is negative, so one of the two roots is positive and the other is negative. In addition, by Vieta's Formulas, the roots sum to 2 n 2\sqrt{n} , so the positive root must be greater than 2 n 2\sqrt{n} in order to produce this sum when added to a negative value. Since 0 < 1 2014 < 2 2014 0 < \frac{1}{\sqrt{2014}} < 2\sqrt{2014} is clearly true, x 2 = 1 2014 x_2 = \frac{1}{\sqrt{2014}} and x 1 + x 3 = 2 2014 x_1 + x_3 = 2\sqrt{2014} . Multiplying these values together, we find that x 2 ( x 1 + x 3 ) = 2 x_2(x_1+x_3) = \boxed{2} .

Upvote if you liked

Moderator note:

Good analysis of this problem, and of determining the intermediate value.

Note: You should check that the other 2 roots are indeed real valued (which is implicit in the problem).

an easier way to solve the last part would have been vietas: x 1 x 2 + x 2 x 3 = 0 x 1 x 3 = 2 2014 x 2 x_1x_2+x_2x_3=0-x_1x_3=\dfrac{2}{\sqrt{2014}x_2} which would take no time. my method was by "oserving"(more of a wierd form of rrt). but your one is nice(+1)

Aareyan Manzoor - 5 years, 5 months ago

Log in to reply

The concern here is to figure out which root is x 2 x_2 . I don't see how much easier your approach is for the very last line of calculation.

Calvin Lin Staff - 5 years, 5 months ago

Log in to reply

Wolfram is showing other two roots are real. I don't know how to prove it any hint.

Department 8 - 5 years, 5 months ago

Log in to reply

Log in to reply

@Calvin Lin That is something I know, I am very stupid.

Department 8 - 5 years, 5 months ago

Log in to reply

@Department 8 Not stupid, just forgetful. Sometimes, when we're concentrating on the harder stuff, we forget the simple stuff. This is why it's important to be grounded in the basics.

Calvin Lin Staff - 5 years, 5 months ago
Ravi Dwivedi
Jan 7, 2016

We observe that 1 2014 \sqrt{\frac{1}{2014}} is a root of given polynomial

Now sum of roots taken two at a time is zero because coefficient of x x is zero,i,e, x 1 x 2 + x 2 x 3 + x 3 x 1 = 0 x_1x_2+x_2x_3+x_3x_1=0 and also product of roots = 2 = x 1 x 2 x 3 -2=x_1x_2x_3 which is negative

This means there is exactly one negative root and two positive roots.

Since the sum of roots = 4029 2014 \frac{4029}{\sqrt{2014}}

This means that the positive root other than 1 2014 \frac{1}{\sqrt{2014}} is greater otherwise the sum of roots cannot be greater than 2 2014 \frac{2}{\sqrt{2014}}

So x 2 = 1 2014 x_2=\frac{1}{\sqrt{2014}}

Now x 1 x 2 + x 2 x 3 + x 3 x 1 = 0 x_1x_2+x_2x_3+x_3x_1=0

x 2 ( x 1 + x 3 ) = x 1 x 3 = x 1 x 2 x 3 x 2 = 2 2014 1 2014 = 2 \implies x_2(x_1+x_3)=-x_1x_3=\frac{-x_1x_2x_3}{x_2}=\frac{\frac{2}{\sqrt{2014}}}{\frac{1}{\sqrt{2014}}}=\boxed{2}

0 pending reports

×

Problem Loading...

Note Loading...

Set Loading...